Solving homogeneous system involving decimal eigenvalues

Click For Summary
The discussion focuses on finding the general solution for a homogeneous system represented by the matrix [3 5; -1 -2]. The eigenvalues are calculated as λ = 1/2 + (√5)/2 and λ = 1/2 - (√5)/2 using the determinant method. The user struggles with reducing the matrix to find the corresponding eigenvectors, initially misplacing parentheses in the matrix setup. After correcting the matrix, the next step involves row reduction, which should yield a row of zeroes, allowing for the extraction of the eigenvector. Proper row reduction will lead to the solution of the system.
maxfails
Messages
10
Reaction score
0

Homework Statement



I need to find the general solution of the system
[3 5]
[-1 -2]

Homework Equations



so to get the eigenvalues, det(A - λI)

The Attempt at a Solution



determinant is (3-λ)(-2-λ) + 5

which would be λ2 - λ - 1

so by the quadratic equation the eigenvalues are λ = 1/2 + (√5)/2 and 1/2 - (√5)/2

but now I don't know how to reduce the matrix to get the eigenvectors?

I think the matrix for the first eigenvalue would be
[3-(1/2 + √5)/2) 5]
[-1 -2-1/2+(√5)/2]

What to do now?
 
Physics news on Phys.org
maxfails said:

Homework Statement



I need to find the general solution of the system
[3 5]
[-1 -2]

Homework Equations



so to get the eigenvalues, det(A - λI)

The Attempt at a Solution



determinant is (3-λ)(-2-λ) + 5

which would be λ2 - λ - 1

so by the quadratic equation the eigenvalues are λ = 1/2 + (√5)/2 and 1/2 - (√5)/2

but now I don't know how to reduce the matrix to get the eigenvectors?

I think the matrix for the first eigenvalue would be
[3-(1/2 + √5)/2) 5]
[-1 -2-1/2+(√5)/2]

What to do now?
Do what you would normally do - row reduce the matrix, but you'll need to fix some misplaced parentheses first.

The matrix should be
[3- (1/2 + √5/2) 5]
[-1 -2- (1/2+√5/2)]

=

[5/2 - √5/2) 5]
[-1 -5/2 - √5/2]

You had a few too many parentheses in your matrix.

When the matrix is row-reduced, there should be a row of zeroes, and you can use the other row to get your eigenvector.
 
Question: A clock's minute hand has length 4 and its hour hand has length 3. What is the distance between the tips at the moment when it is increasing most rapidly?(Putnam Exam Question) Answer: Making assumption that both the hands moves at constant angular velocities, the answer is ## \sqrt{7} .## But don't you think this assumption is somewhat doubtful and wrong?

Similar threads

  • · Replies 2 ·
Replies
2
Views
2K
Replies
5
Views
2K
Replies
8
Views
3K
  • · Replies 2 ·
Replies
2
Views
1K
  • · Replies 7 ·
Replies
7
Views
2K
Replies
5
Views
2K
  • · Replies 8 ·
Replies
8
Views
2K
  • · Replies 7 ·
Replies
7
Views
1K
  • · Replies 10 ·
Replies
10
Views
2K
Replies
2
Views
2K